Series and parallel circuits Flashcards
what is a series circuit?
a complete circuit with more than 1 load and only one pathway.
voltage law
total of all voltage drops is equal to the applied voltage. basically all the voltage is used.
what is another name for voltage drop
IR drop because the voltage drop is equal to the amps*resistance
its important to do what before measuring resistance?
disconnect the load from the circuit
why measure voltage drop instead of resistance
1) you can only test resistance with a disconnected component and resistance changes with current.
2) voltage drop is dynamic, takes heat into consideration.
what are some automotive applications of voltage drop
blower motor, dash lights
Parallel circuits
a parallel circuit is a complete circuit with more than one path for the current to follow.
does an open in one branch of a parallel circuit cause the other branches to stop working
no
Counterintuitively the resistance increases and the current will decrease. Its like the electricity knows what loads are ahead in the circuit and sends less current for less load.
1/Rt = 1/4 + 1/8 + 1/16 Rt = 2.28 I = 12/2.28 I = 5.26amps
1/Rt = 1/4 + 1/8 Rt = 2.66 I = 12/2.66 I = 4.51 amps
parallel circuit law 1 current
not all of the current flows through each branch.
parallel circut law 2 voltage
voltage is the same for each branch
parallel circuit law 3
the sum of the current in each leg equals the total current
how can you measure the total current in a parallel circuit?
treat each leg as a simple circuit. the total current is the sum of all the legs.
the total resistance of a parallel circuit will be more/less than the smallest resistance in any leg?
less
what is kirchoffs voltage law?
the total of all voltage drops is equal to applied voltage.
what would current do if voltage was doubled in a circuit?
E=IR I=E/R 6=12/2 12=24/2 current doubles
what would current do if resistance was doubled in a circuit?
I=E/R
6=12/2
3=12/4
current would be halved
What is the forumula for voltage drop?
Ohm’s law
E=IR
IR drop
Why is the total resistance of a parallel circuit less than the smallest resistance?
1/Rt = 1/R1 + 1/R2
I don’t really get it. More answers here:https://physics.stackexchange.com/questions/269492/why-is-the-resistance-in-a-parallel-circuit-smaller-than-the-resistance-in-a-ser
Why are parallel circuits used instead of series circuits in most automotive applications?
If one leg of a parallel circuit looses continuity the other legs will still work. Dont have a great understanding about this one, need to do more digging.
Kirchoffs current law
the current flowing into a junction is the same as the current flowing out of a junction
What would be the efect of an open circuit in one leg of a parallel portion of a series-parallel circuit?
only the leg with the open would be effected.
What would be the effect of an open circuit in the series portion of a series-parallel circuit?
whole thing would stop working
the amperage in a series circuit is the same or different ?
the same everywhere
the sum of votage drops equals
applied voltage
what is the formula for finding current
E=IR
I=E/R
A series circuit has three reistsors of 4ohms each. The voltage drop across each resistor is 4v. Tech A says the source voltage is 12v. Tech B says the total resitance is 18ohms. Which is correct?
kirchoffs voltage law. the applied voltage is equal to the sum of all voltage drops. 4v*3=12v so source voltage is 12v. Total resistance in a series circuit is the sum of all resistances so the total resistance is 12ohms not 18. TECH A.
a series circuit has two 10ohm bulbs. a third bulb is added in series. Tech A says that the three bulbs will be dimmer than when only two bulbs were in the circuit. Tech B says that the current will increase. Which tech is correct?
Three bulbs will be dimmer than two because there is more resistance to flow and therefore less current.
I=E/R
- 6amps = 12v/20ohms
- 4amps = 12v/30ohms
Tech A says that the sum of the voltage drops in a series circuit should equal the source voltage. Tech B says that the current varies depending on the value of resitance in a series circuit.
Both techs are correct.
Kirchoffs voltage law states that applied voltage is equal to the sum of voltage drops. Therefore Tech A is correct.
Increased resistance decreases flow (current) I=E/R 0.6amps = 12v/20ohms 0.4amps = 12v/30ohms Therefore Tech B is correct.
Two bulbs are connected in parallel to a 12v battery. One has 6ohm resitance the other 2ohm resistance. Which bulb will be brighter?
Resistance decreases flow (current) and current is brightness. Therefore the 2ohm bulb will be brighter.
Calculate the total resistance and current in a parallel circuit with three resistances, 4ohms, 8ohms, and 16ohms. Assume 12v?
1/Rt = 1/4 + 1/8 + 1/16 1/Rt = .4375 Rt = 1/.4375 Rt = 2.28ohms
E=IR
I=E/R
I=12/2.28
I = 5.26amps
A vehicle has four parking lights in parallel, one of them blows. Tech A says that this could cause the fuse to blow. Tech B says that the current would decrease.
Increased resistance decreases flow (amps). Therefore current would increase. If current increased beyond the rating of the fuse the fuse could blow. Therefore Tech A is correct.
1/Rt = 1/2 + 1/2+ 1/2 +1/2 1/Rt = 2 Rt = 1/2 Rt = 0.5ohms
1/Rt = 1.5 Rt = 1/1.5 Rt = 0.6ohms